Reversing Order of Integration for Double Integral Problem

Click For Summary
The discussion centers on the challenges of reversing the order of integration in a double integral problem. Initial attempts at finding the anti-derivative were incorrect, particularly confusing the function forms, leading to the wrong conclusion about limits and anti-derivatives. The correct anti-derivative for 1/(1+y^2) is arctan(y) + C, not a logarithmic function. Participants agree that reversing the order of integration simplifies the problem significantly. Ultimately, using tools like Wolfram can aid in resolving complex integration challenges.
emelie_earl
Messages
3
Reaction score
0
8yv6i.png


My idea was that the limits are
BrQKm.png


and that the anti-derivative of dy was

xlog(1+y^2)

but that seems wrong...

maybe use these limits instead
YKvJd.png


and start with dx?

gives us
rDFwG.png


then we take dy

rgKWx.png


guess, i figured it out eventually with the help of wolfram with the last integration
 
Last edited:
Physics news on Phys.org
The reason your 1st attempt didn't work is that \int\frac{1}{1+y^2}\,dy=\tan^{-1}(y)+C\,.
 
emelie_earl said:
8yv6i.png








My idea was that the limits are
BrQKm.png


and that the anti-derivative of dy was

xlog(1+y^2)

but that seems wrong...
Yes, it is wrong. It is standard "Calculus I" mistake to treat a function of the variable as if it were just the variable but you should be past that by the time you are doing multiple integrals. "1/(1+ y^2)" is NOT the same as 1/y and its anti-derivative is not a logarithm. The anti-derivative of 1/(1+ y^2) is arctan(y)+ C. That's a standard anti-derivative that you should have memorized.

maybe use these limits instead


YKvJd.png


and start with dx?

gives us
rDFwG.png


then we take dy

rgKWx.png


guess, i figured it out eventually with the help of wolfram with the last integration
Yes, reversing the order of integration is the best way to handle this one. Integerating arctan(1)- arctan(x^2)= \pi/4- arctan(x^2) is likely to be very difficult!
 
HallsofIvy said:
...
Yes, reversing the order of integration is the best way to handle this one. Integerating arctan(1)- arctan(x^2)= \pi/4- arctan(x^2) is likely to be very difficult!
Yes. I agree !
 
Question: A clock's minute hand has length 4 and its hour hand has length 3. What is the distance between the tips at the moment when it is increasing most rapidly?(Putnam Exam Question) Answer: Making assumption that both the hands moves at constant angular velocities, the answer is ## \sqrt{7} .## But don't you think this assumption is somewhat doubtful and wrong?

Similar threads

  • · Replies 17 ·
Replies
17
Views
4K
  • · Replies 9 ·
Replies
9
Views
2K
Replies
2
Views
2K
  • · Replies 4 ·
Replies
4
Views
2K
  • · Replies 1 ·
Replies
1
Views
2K
Replies
3
Views
2K
  • · Replies 4 ·
Replies
4
Views
2K
Replies
2
Views
1K
Replies
4
Views
3K
Replies
4
Views
2K